LSAT and Law School Admissions Forum

Get expert LSAT preparation and law school admissions advice from PowerScore Test Preparation.

User avatar
 Dave Killoran
PowerScore Staff
  • PowerScore Staff
  • Posts: 5853
  • Joined: Mar 25, 2011
|
#85632
Complete Question Explanation

(The complete setup for this game can be found here: viewtopic.php?f=386&t=3729)

The correct answer choice is (B)

According to the third rule, K > S, and so answer choice (B) cannot be true and is correct. In that
answer, S would be first, K second, and F third, causing the violation.
 wwarui
  • Posts: 32
  • Joined: Nov 13, 2011
|
#13251
Hello:

Please explain or illustrate why B is the answer.

How can K be second when the stimulus states that Ron chose the knife before the spoon? In any event, isn't KFS before wj OR mp before wj? I'm obviously missing something.
Thanks!!
 Emily Haney-Caron
PowerScore Staff
  • PowerScore Staff
  • Posts: 577
  • Joined: Jan 12, 2012
|
#13265
Hello,

I think it is possible you missed that the question is asking for the answer that CANNOT be an accurate list of the objects. B cannot be an accurate list for the reason you stated: the knife needs to be selected before the spoon, and since the utensils all need to be washed consecutively, answer B would require that the spoon be washed first - which would violate the rules. The other answers could all occur without violating a rule.

Does that answer your question?
 wwarui
  • Posts: 32
  • Joined: Nov 13, 2011
|
#13268
Hi Emily:

YES! I see it now. I often do this and it costs me a lot of time trying to figure out what I do not understand. In this case I tried so many times before asking for assistance.

Thanks!!!!

Get the most out of your LSAT Prep Plus subscription.

Analyze and track your performance with our Testing and Analytics Package.